LSAT and Law School Admissions Forum

Get expert LSAT preparation and law school admissions advice from PowerScore Test Preparation.

 Administrator
PowerScore Staff
  • PowerScore Staff
  • Posts: 8917
  • Joined: Feb 02, 2011
|
#33167
Complete Question Explanation
(See the complete passage discussion here: lsat/viewtopic.php?t=13636)

The correct answer choice is (C)

Again, the answers to Global Reference Must Be True questions are quite difficult to prephrase, so the Fact Test must be applied to find the choice that can be confirmed by the information provided in the passage.

Answer choice (A): The passage provides that prions occur naturally in cell tissues, and that they possess the capability to, on their own, transform themselves into abnormal shapes. This choice discusses the transmission of CJD from an infected individual to an uninfected one; this answer is not confirmed by the information provided in the passage, so it fails the Fact Test and cannot be the right answer to this Must Be True question.

Answer choice (B): If you found this answer choice appealing, that is, of course, by design; the author does mention in the final paragraph that other neurological disorders are now thought to be caused by similar processes, but this choice goes much farther than that, claiming that most infectious diseases are now thought to be caused by prions, This is entirely unsupported by the passage, so it should be ruled out of contention.

Answer choice (C): This is the correct answer choice. In the third paragraph, the author mentions that prions are able to reproduce in an unexpected way, helping to create the pathogenic conformation.

Answer choice (D): The author does mention at the end of the passage that those common neurological conditions are now thought to be caused by processes similar to those that cause CJD, but this does not mean that those disorders are different conformations of the CJD-causing pathogenic prion.

Answer choice (E): The author never makes the assertion presented in this answer choice, nor does the passage even provide such a comparison, so this choice cannot be the right answer to this Must Be True question.
 Nikki Siclunov
PowerScore Staff
  • PowerScore Staff
  • Posts: 1362
  • Joined: Aug 02, 2011
|
#28296
We recently received the following question from a student:
I initially ruled out (D) very swiftly and almost picked the correct answer (C), but then went back and justified (D) by this part of the passage from line 52, near its ending: "Furthermore it is now believed that a similar process of protein malformation may be involved in other..... neuro conditions such as Alzheimers and Parkinsons..."

Is (D) wrong because it goes too far in degree/ is too absolute? Too certain in its declaration that Alzehimers and Parkinsons "ARE" caused by a similar thing?
Thanks for your question! :) The difference between answer choices (C) and (D) is subtle, but important. It all comes down to the issue of provability: which of the two statements is more strongly supported by the information contained in the passage? Although the phrase “most strongly supported” exacts a somewhat lower standard of scrutiny than the classic “which one of the following can be properly inferred…” stem, the correct answer to that question must still be very strongly suggested or implied by the stimulus, even if it's not 100% provable by it. That said, this is not an issue with this particular MBT question: answer choice (C) DOES state an undeniably provable claim. Answer choice (D) does not. Here's why:

In the third paragraph, the author mentions that prions are able to reproduce in an unexpected way, helping to create the pathogenic conformation (lines 30-36). Additionally, infection is said to occur "only if the pathogenic agent is able to reproduce itself in the host organism" (lines 3-5). Thus, it is reasonable to conclude that if abnormally shaped prions couldn't reproduce themselves, then they would not exhibit the infectious properties described in the third paragraph, and would not cause CJD. Consequently, answer choice (C) states a provable claim and is correct.

By contrast, we have no evidence to substantiate the claim made in answer choice (D). Are Alzheimer's and Parkinson's disease caused by different conformations of the same prion that caused CJD? Who knows! All we know is that a similar process of protein malformation may be responsible for these diseases. It's entirely possible that a different prion was able to reproduce itself in a similar way: it is the process that is similar, not the pathogenic agent.

Hope this clears it up!

Thanks,
 cecilia
  • Posts: 66
  • Joined: Nov 07, 2011
|
#28319
Thanks Nikki!
 glasann
  • Posts: 61
  • Joined: Jan 07, 2020
|
#76632
I incorrectly chose answer E (which states that "prion diseases generally progress more aggressively than other diseases caused by other known pathogens") because of line 39.

Line 39 states, "because prions, unlike other pathogens, occur naturally in the body as proteins, the body does not produce an immune response when they are present."

How does that not let us infer what they put in answer E? Is it because E is specifically comparing the aggressiveness to other diseases, which really isn't mentioned around line 39?

Thanks!
 Jeremy Press
PowerScore Staff
  • PowerScore Staff
  • Posts: 1000
  • Joined: Jun 12, 2017
|
#77110
Hi glasann,

Yes, that's basically it! Also, push a little further with that reference you found, where it says, "CJD is inevitably fatal, though there are wide variations in pre-symptomatic incubation times and in how aggressively the disease progresses." Since there is wide variation in aggressiveness of one particular prion disease (CJD), it follows that (without more information) we won't be able to support a general claim that prion diseases are "generally...more aggressive" than other pathogen diseases.

I hope this helps!

Jeremy

Get the most out of your LSAT Prep Plus subscription.

Analyze and track your performance with our Testing and Analytics Package.